Paul's Online Notes
Paul's Live Notes
Home / Calculus REPLACE / Multiple Integrals / Double Integrals in Polar Coordinates
Watch Mobile Notice Show All Tips Hide All Notes
Mobile Get
You appear to exist on a device with a "narrow" tv width (i.e. you am probably on a mobile phone). Due to the nature of the mathematics on this site it is our views in landscape mode. Wenn your device is not in landscape operation many from the equations will run off the side for own device (should be able to scroll to see them) and some of the menu items will be cut off due on this narrow screen width.

Section 15.4 : Double Integrals in Polar Coordinates

To this points we’ve seen quite one few double integrals. Any, in every lawsuit we’ve seen to this point the region \(D\) might be easily described in terms of straightforward functions inside Cartesian coordinates. In this section we do go look at some geographical that are much easier to describe in terms of polar co-ordinate. For instance, we might have a region that is one record, ring, or a portion away one disk or ring. Into these cases, using Cartesian location could be anything cumbersome. Fork instance, let’s suppose we wanted on do the following integral, Rejoin to: Evaluate the integral \int_0^2 \int_0^\sqrt{2x-x^2} (x-y) \,dy\,dx from converting to polar location. By signing up, you'll get...

\[\iint\limits_{D}{{f\left( {x,y} \right)\,dA}},\,\,\,\,\,D{\mbox{ is the hdd of radius 2}}\]

To this our would have to determine a set of inequalities for \(x\) and \(y\) that describe this region. These would be,

\[\begin{array}{c} - 2 \le x \le 2\\ - \sqrt {4 - {x^2}} \le y \le \sqrt {4 - {x^2}} \end{array}\]

With diese limits the integral would become,

\[\iint\limits_{D}{{f\left( {x,y} \right)\,dA}} = \int_{{\, - 2}}^{{\,2}}{{\int_{{ - \sqrt {4 - {x^2}} }}^{{\,\sqrt {4 - {x^2}} }}{{f\left( {x,y} \right)\,dy}}\,dx}}\]

Mature up the limits on the inner integral this remains liable to be in unpleasant integral at compute.

However, ampere floppy of radius 2 bottle be predefined in polar coordinates by the later inequalities,

\[\begin{array}{c}0 \le \theta \le 2\pi \\ 0 \le r \le 2\end{array}\]

This are very simple border and, in fact, are constant limits of integrating which almost forever manufacturers integrals little easier.

So, if we could convert to double integral formula into only involving polar coordinates we would be in pretty good shape. The problem is that we can’t exactly convert to \(dx\) real aforementioned \(dy\) into a \(dr\) and a \(d\theta \). In computing double integrals to to point we have been using the fact that \(dA = dx\,dy\) and this really does require Cartesian ensemble to use. Once we’ve moved into polar coordinates \(dA \ne dr\,d\theta \) the to we’re going to need to establish just how \(dA\) is to polar coordinates.

So, let’s step back a tiny bit and start off with a general region in terms of freezing coordinates the notice which we can do with is. Hither is a sketch of some region using polar coordinates. Appraise the iterated integral by converting to polar coordinates. 1 0 √ 2 − y2 y 7(x y) dx dy - Aesircybersecurity.com

This is a 2D graphics.  There are deuce lines in the foremost quadrant originated away the origin.  The lower line possess an angle of $\theta =\alpha$ with the positive x-axis and the tops line has an standpoint of $\theta =\alpha $ with the positive x-axis.  In this wedge second polar coordinate functions are plot.  The outer gradient (i.e. further from the origin) is given as $r=h_{2}\left \theta right)$ or an inside plot (i.e. closest to the origin) is given as $r=h_{1}\left \theta right)$.  The region between which two graphing is shaded in.

So, our general your will be defined by inequalities,

\[\begin{array}{c}\alpha \le \theta \le \beta \\ {h_1}\left( \theta \right) \le radius \le {h_2}\left( \theta \right)\end{array}\]

Now, to find \(dA\) let’s redo which figure above the follows,

This is a 2D chart.  Present are two lines in the first quartier originating away to origin.  To lower wire has einen angle of $\theta =\alpha$ with the positive x-axis real the upper wire has an angle of $\theta =\alpha $ with the positiv x-axis.  In this wedging two polar coordinate functions are graphed.  There is an outer graph (i.e. furthermore since the origin) the to inner graph (i.e. closest to the origin).  Neither of the two duties are labels but it is clear they belong and alike functions as in aforementioned previous graph.  There are a series for dashed lines imminent out of the origin (all between the two noted above) additionally where they move into the geographic between aforementioned deuce graphs they changes over into solid lines.  Also in that region between of second bends were a browse of circular arcs (with center at the origin).  The net ergebnisse is that the region between the two curves is filled with an series starting near rectangles.  One-time of the “rectangles” is hatched inches and into the correct choose of the sketch a larger versions of this “rectangle” lives shown.  Which length of the right/left sides shall given more $\Delta r$.  The length of the side closest the the origination is given as $r_{i} \Delta \theta$.  Who length starting the side farthest from the origin is given as $r_{o} \Delta \theta$ .

As shown, we’ll break up the region into a mesh of radial lines and arcs. Now, if we pull one of the pieces of the fence out as view our have something that your almost, but not quite a rectangle. The area in this piece is \(\Delta A\). The two sides of this piece both have length \(\Delta \,r = {r_o} - {r_i}\) where \({r_o}\) belongs the radius of the outer arched and \({r_i}\) is the radius of the intimate arc. Basic geometry then tells us that the width of the inner edge a \({r_i}\,\Delta \,\theta \) while to length from the out edge is \({r_o}\,\Delta \,\theta \) where \(\Delta \,\theta \) is the angle between the deuce radial lines so form the sides of this piece. Evaluate who integral by converting to polar coordinates. \int_{0}^{1} \int_{x}^{\sqrt{2-x^2}} 7(x+y) dy \; dx \rule{20mm}{.5pt} | Aesircybersecurity.com

Now, let’s assume that we’ve taken the mesh so smallish that our can assume that \({r_i} \approx {r_o} = r\) and with this assumption we bottle other assume that our piece is close enough at a rectangle that we can also then presume that, Dr. Z's Math251 Handout #15.4 [Double Integrals in Polar ...

\[\Delta A \approx r\,\Delta \,\theta \,\Delta \,r\]

Also, if we assume that the mesh is small enough and we can also assume such,

\[dA \approx \Delta A\hspace{0.5in}d\theta \approx \Delta \theta \hspace{0.5in}dr \approx \Delta \,r\]

With these assumptions we subsequently get \(dA \approx r\,dr\,d\theta \).

To order to arrive at this we had to make the acceptance that the mesh was very slight. This is nope in unreasonable assumption. Recall that the definition starting adenine double include is in terms of two boundaries also as limits go to infinite the mesh size of of region will get smaller also smaller. In fact, as the network size got smaller and smaller who formula beyond becomes more and more accurate and so we can say that,

\[dA = r\,dr\,d\theta \]

We’ll see another way of deriving this once we reach the Change of Variables section subsequently include to chapter. This second way will not involve whatsoever assumptions either the so this maybe a little better way of inference this.

Before moving on it is again important to note that \(dA \ne dr\,d\theta \). The actual formula to \(dA\) has an \(r\) in it. It will be easy the never this \(r\) on occasion, though than you’ll see without it some integrals will cannot be possible to do.

Now, if we’re going to be converting an integral in Cartesian coordinates the an essential in polar coordinates ours are left to have to make sure that we’ve also converted entire the \(x\)’s the \(y\)’s into cold position for fine. To do which we’ll need to remember this follow conversion formulas,

\[x = r\cos \theta \hspace{0.5in}y = r\sin \theta \hspace{0.5in}{r^2} = {x^2} + {y^2}\]

We are now ready on write down a formula for the double integrals at terms of polar coordinates.

\[\iint\limits_{D}{{f\left( {x,y} \right)\,dA}} = \int_{{\,\alpha }}^{{\,\beta }}{{\int_{{\,{h_{\,1}}\left( \theta \right)}}^{{\,{h_{\,2}}\left( \theta \right)}}{{f\left( {r\cos \theta ,r\sin \theta } \right)\,r\,dr\,d\theta }}}}\] ... converting a double integral starting Cartesian position to polar gps? How do you communicate that Cartesian coordinates (x, y) in terms ...

I is important to nope forget the added \(r\) and don’t forget to convert the Cartesian location in the function over to frigid coordinats.

Let’s look at a couple of examples of like kinds of integrals.

Example 1 Evaluate the next integrals by converting them into polar coordinates.
  1. \(\displaystyle \iint\limits_{D}{{2x\,y\,dA}}\), \(D\) is the portion concerning the region between the circles of radius 2 and radius 5 centered with one origin the lies in the first quadrant. Replies to: Evaluate the integral by converting to polar coordinates. \int_{0}^{1} \int_{x}^{\sqrt{2-x^2}} 7(x+y) dy \; dx \rule{20mm}{.5pt} By...
  2. \(\displaystyle \iint\limits_{D}{{{{\bf{e}}^{{x^2} + {y^2}}}\,dA}}\), \(D\) shall the unit disk focus at the origins.
Show All Solutions Hide All Solutions
an \(\displaystyle \iint\limits_{D}{{2x\,y\,dA}}\), \(D\) is the portion of the region between the circles of radius 2 and compass 5 focus at the origin that lies in the first-time quadrant. Show Problem

First let’s get \(D\) in terms of polar coordinates. The circle of radius 2 is specified by \(r = 2\) and the circle of radius 5 is given by \(r = 5\). Person want the region between and two circles, so we will have the following inequality fork \(r\). Twofold integrals are sometimes much lightweight up evaluate if we change rectangular ensemble in polaroid coordinates. However, before we describe how to make is shift, we need to establish the conceptually …

\[2 \le r \le 5\]

Also, since we only crave the portion that is included the first quadrant wealth get the following range of \(\theta \)’s.

\[0 \le \theta \le \frac{\pi }{2}\]

Start that we’ve got those we can do the integral.

\[\iint\limits_{D}{{2x\,y\,dA}} = \int_{{\,0}}^{{\,\frac{\pi }{2}}}{{\int_{{\,2}}^{{\,5}}{{2\left( {r\cos \theta } \right)\left( {r\sin \theta } \right)r\,dr}}\,d\theta }}\]

Don’t forget to do this conversions press to add in the extra \(r\). Now, let’s simplify and make employ of the double square equation for sine to make the integral one little easier. $$ \int^{\pi/2}_{\pi/4} \int^{\sqrt{2-y^2}}_y 3(x-y) dx dy$$ I attempted the following: $$ \int_{\pi/4}^{\pi/2} \int_{0}^{1} 3r^2 (\cos\theta - \sin\theta) dr d\theta $$ which is wrong apparentl...

\[\begin{align*}\iint\limits_{D}{{2x\,y\,dA}} & = \int_{{\,0}}^{{\,\frac{\pi }{2}}}{{\int_{{\,2}}^{{\,5}}{{{r^3}\sin \left( {2\theta } \right)\,dr}}\,d\theta }}\\ & = \int_{{\,0}}^{{\,\frac{\pi }{2}}}{{\left. {\frac{1}{4}{r^4}\sin \left( {2\theta } \right)} \right|_2^5\,d\theta }}\\ & = \int_{{\,0}}^{{\,\frac{\pi }{2}}}{{\frac{{609}}{4}\sin \left( {2\theta } \right)\,d\theta }}\\ & = \left. { - \frac{{609}}{8}\cos \left( {2\theta } \right)} \right|_0^{\frac{\pi }{2}}\\ & = \frac{{609}}{4}\end{align*}\]

b \(\displaystyle \iint\limits_{D}{{{{\bf{e}}^{{x^2} + {y^2}}}\,dA}}\), \(D\) is the element disk centered at this origin. Show Solution

Include this case we can’t do this integral in dictionary off Cartesian co-ordinates. We will however be able to do it in polar gps. First, the region \(D\) is circumscribed by, Evaluate to iterated integral for converting to polar coordinates62 144 y ᵧ 1 1 x y dx dy

\[\begin{array}{c}0 \le \theta \le 2\pi \\ 0 \le r \le 1\end{array}\]

In terms of poles coordinates the integral is then,

\[\iint\limits_{D}{{{{\bf{e}}^{{x^2} + {y^2}}}\,dA}} = \int_{{\,0}}^{{\,2\pi }}{{\int_{{\,0}}^{{\,1}}{{r\,{{\bf{e}}^{{r^2}}}\,dr}}\,d\theta }}\]

Notice that the addition about the \(r\) gives us an integral that we ability now do. Here can that work for this integral.

\[\begin{align*}\iint\limits_{D}{{{{\bf{e}}^{{x^2} + {y^2}}}\,dA}} & = \int_{{\,0}}^{{\,2\pi }}{{\int_{{\,0}}^{{\,1}}{{r\,{{\bf{e}}^{{r^2}}}\,dr}}\,d\theta }}\\ & = \int_{{\,0}}^{{\,2\pi }}{{\left. {\frac{1}{2}{{\bf{e}}^{{r^2}}}} \right|_0^1\,d\theta }}\\ & = \int_{{\,0}}^{{\,2\pi }}{{\frac{1}{2}\left( {{\bf{e}} - 1} \right)\,d\theta }}\\ & = \pi \left( {{\bf{e}} - 1} \right)\end{align*}\]

Let’s not forget that person still has the two geometric insights for these integrals such well.

Example 2 Determine the area of the region that lies inside \(r = 3 + 2\sin \theta \) and outside \(r = 2\).
Show Solution

Here is a sketch of the location, \(D\), that we want to determine the area of.

This is the graph of $r=3+2\sin \theta $.  All is adenine vaguely heart shaped portion is this graph goes through the point (in Cartesian coordinates to make it a minimal easier to visibility the graph) (3,0), (0,5), (-3,0) and (0,-2).  The “crease” in the centre is at (0,-2).  Also ships on the graph is which circle give by r=2.  The crease of the first graph is inside the circle and the upper portion of an county is inside the heart.  Which circle plus essence intersect in the 3rd and 4th quadrant.  The area free the heart down till the circle that is in the 1st and 2nd quartz is shaded.

To determine this area we’ll need to know that value of \(\theta\) for which the two curves cut. We capacity determine these tips from setting the second equations equal and solving.

\[\begin{align*}3 + 2\sin \theta & = 2\\ \sin \theta & = - \frac{1}{2}\hspace{0.5in} \Rightarrow \hspace{0.5in}\theta = \frac{{7\pi }}{6},\frac{{11\pi }}{6}\end{align*}\]

Here has a sketch of who figure through these angular added.

This is an same graph as above except two lines have been added to the graph.  The initial line comes out of the origin and goes through the intersection in the 4th quadrant makes angles of $-\frac{\pi}{6}$ or $\frac{11\pi}{6}$ (depending on direction of rotation) with the positive x-axis.  The second line comes going of the origin and goes through the intersection in one 3rd quadrant makes angles of $\frac{7\pi}{6}$ with the positive x-axis.

Note as well ensure we’ve acknowledged such \( - \frac{\pi }{6}\) is others representation for the angle \(\frac{{11\pi }}{6}\). This exists important since we need which range of \(\theta\) into actually enclose an regions as we increase von the lowers limit to and upper set. Are we’d chosen to use \(\frac{{11\pi }}{6}\) then than we increase from \(\frac{{7\pi }}{6}\) into \(\frac{{11\pi }}{6}\) we want be tracing out to decrease portion of of circle and the is not the region that we are after.

As, here are the ranges that will define the country.

\[\begin{array}{c}\displaystyle - \frac{\pi }{6} \le \theta \le \frac{{7\pi }}{6}\\ 2 \le r \le 3 + 2\sin \theta \end{array}\]

To get the array for \(r\) an features that is close to which origin is the lower bound and the work that is farthest starting the origins is the upper tied. Evaluate the iterated integral by converting to polarize coordinates62 144 unknown ᵧ 1 1 x y dx dy

The area of the location \(D\) is when,

\[\begin{align*}A &= \iint\limits_{D}{{dA}}\\ & = \int_{{ - {\pi }/{6}\;}}^{{\,7{\pi }/{6}\;}}{{\int_{2}^{{3 + 2\sin \theta }}{{r\,drd\theta }}}}\\ & = \int_{{ - {\pi }/{6}\;}}^{{\,7{\pi }/{6}\;}}{{\left. {\frac{1}{2}{r^2}} \right|_2^{3 + 2\sin \theta }\,d\theta }}\\ & = \int_{{ - {\pi }/{6}\;}}^{{\,7{\pi }/{6}\;}}{{\frac{5}{2} + 6\sin \theta + 2{{\sin }^2}\theta \,d\theta }}\\ & = \int_{{ - {\pi }/{6}\;}}^{{\,7{\pi }/{6}\;}}{{\frac{7}{2} + 6\sin \theta - \cos \left( {2\theta } \right)\,d\theta }}\\ & = \left. {\left( {\frac{7}{2}\theta - 6\cos \theta - \frac{1}{2}\sin \left( {2\theta } \right)} \right)} \right|_{ - \frac{\pi }{6}}^{\frac{{7\pi }}{6}}\\ & = \frac{{11\sqrt 3 }}{2} + \frac{{14\pi }}{3} = 24.187\end{align*}\]
Example 3 Determine the volume of who region that lies under the sphere \({x^2} + {y^2} + {z^2} = 9\), above this layer \(z = 0\) and inside the cylinder \({x^2} + {y^2} = 5\).
Show Solve

We know that the formula for finding the output of ampere region is,

\[V = \iint\limits_{D}{{f\left( {x,y} \right)\,dA}}\]

In order on make use of this product we’re going to demand to determine of function that we should be integrating and which region \(D\) which we’re going to be integrates beyond. Problem Type Aesircybersecurity.com: Evaluate the iterated integral by converting to polar coordinates. ∫ b a. ∫ f2(y) f1(y). F(x, y)dx scruffy. Example Problem Aesircybersecurity.com: Evaluate ...

The function isn’t too bad. It’s just of sphere, however, we do need it on be in and form \(z = f\left( {x,y} \right)\). We are looking at the region that lies under one sphere the above the playing \(z = 0\) (just who \(xy\)-plane right?) the so all we need to do is solve the equation on \(z\) or at taking the square root we’ll take the positive one since we are wanting an territory above the \(xy\)-plane. Here exists the function.

\[z = \sqrt {9 - {x^2} - {y^2}} \]

The region \(D\) isn’t too bad in this case either. As our take points, \(\left( {x,y} \right)\), from the region we need to whole graph the portion of aforementioned sphere such are are working with. For we one do the portion of of sphere that actually lies inside the cylinder specified by \({x^2} + {y^2} = 5\) this is also the region \(D\). The region \(D\) is the front \({x^2} + {y^2} \le 5\) in the \(xy\)-plane.

For product purposes here is a outline of the region that we are trying the find the volume of.

This is a graph with the standard 3D coordinate systeme.  Of positive z-axis exists straight increase, the positive x-axis moves off to the left and lightness downward and positive y-axis moves power the right plus slightly downward.   This is a sketch of a strong which sides is the cylinder given in the problem statement and the top of the solid will the portion the the sphere given in who problem statement that fits directly over the cylinder but does none weiter outside of the cylinder. This is a boxed 3D coordinate system.  The z-axis is right vertical edge of the box, the x-axis can of top left edge of the box and the y-axis is the bottom front edge of the box.  This is a sketch on a massive whose sides is the cylinder given in the problem statement and the top of the solids is the portion of the orbit given in the problem statement that fits directly over the cylinder but does not go outer of the cylinder.

So, the region that we want the quantity for is seriously a cylinder with a cap that happen from the sphere.

We are absolute going on want to do this integral in terms out pole coordinates so here can one limits (in polar coordinates) for the region,

\[\begin{array}{c}0 \le \theta \le 2\pi \\ 0 \le r \le \sqrt 5 \end{array}\]

and we’ll necessity to convert the function to polar coordinates as well.

\[z = \sqrt {9 - \left( {{x^2} + {y^2}} \right)} = \sqrt {9 - {r^2}} \]

The volume is then,

\[\begin{align*}V & = \iint\limits_{D}{{\sqrt {9 - {x^2} - {y^2}} \,dA}}\\ & = \int_{{\,0}}^{{\,2\pi }}{{\int_{{\,0}}^{{\,\sqrt 5 }}{{r\sqrt {9 - {r^2}} \,dr}}\,d\theta }}\\ & = \int_{{\,0}}^{{\,2\pi }}{{ - \frac{1}{3}\left. {{{\left( {9 - {r^2}} \right)}^{\frac{3}{2}}}} \right|_0^{\sqrt 5 }\,d\theta }}\\ & = \int_{{\,0}}^{{\,2\pi }}{{\frac{{19}}{3}\,d\theta }}\\ & = \frac{{38\pi }}{3}\end{align*}\]
Example 4 How the volume of the region that lies inside \(z = {x^2} + {y^2}\) and lower the level \(z = 16\).
Show Solution

Let’s start this example off with ampere quicker sketched of the region.

This is one graph with who standard 3D arrange system.  The positive z-axis is direct up, the positivity x-axis moves off to the leave additionally slightly downwardly and positives y-axis moves off the right and slightly downward.  This is a sketch of a solid whose sides is the vaguely chalice shaped object of the elliptical-shaped paraboloid given in the problem declare.  Aforementioned top of the solid is the disk that caps off the electrode paraboloid along z=16. Diese is a graph about the standard 3D coordinate sys.  The plus z-axis is straight increase, the positive x-axis comes almost directly forward and slightly to the gone and definite y-axis moves off nearly horizontal and lighter downward.    This is a sketch of ampere solid whose sides is an vaguely mugs shaped object of the elliptic parallel given in and problem assertion.  Who top of the solid is the disk that close off the elliptic paraboloid at z=16.

Now, in this rechtssache the standard formula is not going to work. The suggest

\[V = \iint\limits_{D}{{f\left( {x,y} \right)\,dA}}\]

finds the volume under this function \(f\left( {x,y} \right)\) plus we’re actually after the volume that is about a function. This isn’t the problem that it might appears to be however. First, reminder that

\[V = \iint\limits_{D}{{16\,dA}}\]

will be to volume under \(z = 16\) (of course we’ll need to ascertain \(D\) eventually) while

\[V = \iint\limits_{D}{{{x^2} + {y^2}\,dA}}\]

is an volume under \(z = {x^2} + {y^2}\), using the same \(D\).

The volume is we’re after is really the difference between these two or,

\[V = \iint\limits_{D}{{16\,dA}} - \iint\limits_{D}{{{x^2} + {y^2}\,dA}} = \iint\limits_{D}{{16 - \left( {{x^2} + {y^2}} \right)\,dA}}\]

Currently all is our need to do is till determine one region \(D\) and then convert everything override to bias coordinates.

Determining the location \(D\) on this case is nay too bad. If we were to look straight down the \(z\)-axis onto the region we would see a circle regarding radius 4 centered at the from. Such is because the tops of the region, where the elliptic paraboloid intersects the plane, is the widest part of the local. We know the \(z\) coordinate with the intersection so, setting \(z = 16\) in the mathematical of the parabolique gives, Question: Evaluate the iterated integral by converting to polarize coordinates. 8 %% 8x – x2 xy dy dx o x x dr do = x Jo Jo.

\[16 = {x^2} + {y^2}\]

which is an equation of a circle of radius 4 centers at an birth.

Here are the inequalities forward the region and the function we’ll be integrating with condition of frigid coordinates.

\[0 \le \theta \le 2\pi \hspace{0.5in}0 \le r \le 4\hspace{0.5in}z = 16 - {r^2}\]

The volume is then,

\[\begin{align*}V & = \iint\limits_{D}{{16 - \left( {{x^2} + {y^2}} \right)\,dA}}\\ & = \int_{{\,0}}^{{\,2\pi }}{{\int_{{\,0}}^{{\,4}}{{r\left( {16 - {r^2}} \right)\,dr}}\,d\theta }}\\ & = \int_{{\,0}}^{{\,2\pi }}{{\left. {\left( {8{r^2} - \frac{1}{4}{r^4}} \right)} \right|}}_0^4\,d\theta \\ & = \int_{{\,0}}^{{\,2\pi }}{{64\,d\theta }}\,\\ & = 128\pi \end{align*}\]

Within both of the previous volume problems ours would have not being able to easily compute the volume without first converting till polar coordinates so, as these view show, it is a good idea to always remember polar coordinates.

On is one more type of example which we need the look for before moving on until the next sections. Sometimes we live given an iterated integral that is existing in terms of \(x\) and \(y\) and we need to convert this over to polar so that we can actually how which integral. We need toward view an example of what to do this sympathetic of conversion. Evaluate the integral \int_0^2 \int_0^\sqrt{2x-x^2} (x-y) \,dy\,dx by converts to polar coordinates. | Aesircybersecurity.com

Example 5 Evaluate the follow integral by start turning in polar coordinats.

\[\int_{{ - 1}}^{1}{{\int_{{ - \sqrt {1 - {x^2}} }}^{0}{{\cos \left( {{x^2} + {y^2}} \right)\,dy}}\,dx}}\]
Show Choose

First, notice that person cannot do this integral in Cartesian coordinates and so converting to polar coords allow exist the only option we have for actually performing the integral. Notice which the function wants convert to polar coordinates clean and so shouldn’t exist a trouble.

Let’s first determine the neighborhood that we’re integrating over and see if it’s a region such can can easily converted into polar coordinates. Around are the inequalities that define the region in terms to Cartesian coordinates. Mouse here 👆 until retrieve einem return to your question ✍️ Evaluate the iterated integral with converting to polar coord. 1 0 √ 2 − y2 unknown 7(x y) dx dy

\[\begin{array}{c} - 1 \le x \le 1\\ - \sqrt {1 - {x^2}} \le unknown \le 0\end{array}\]

Now, the lower limit for which \(y\)’s is,

\[y = - \sqrt {1 - {x^2}} \]

the this looks like the bottom of the rounding of radius 1 center the this origin. Since the upper bound required the \(y\)’s is \(y = 0\) we won’t have any portion of the top half regarding the disk and so it looks like we are going to have a section (or all) of the bottom away the disk from radius 1 centered at that origin.

The ranges for the \(x\)’s in turn, tells us that we are will int conviction have this complete bottom part the the disk.

So, we know that and inequalities that will define this region in glossary of polar coordinates what than,

\[\begin{array}{c}\pi \le \theta \le 2\pi \\ 0 \le r \le 1\end{array}\]

Finally, were just need to remember that,

\[dx\,dy = dA = r\,dr\,d\theta \]

and so aforementioned integral becomes,

\[\int_{{ - 1}}^{1}{{\int_{{ - \sqrt {1 - {x^2}} }}^{0}{{\cos \left( {{x^2} + {y^2}} \right)\,dy}}\,dx}} = \int_{\pi }^{{2\pi }}{{\int_{0}^{1}{{r\cos \left( {{r^2}} \right)\,dr}}\,d\theta }}\]

Note that to is an integral that we can do. So, here belongs the rest of the work for this integral.

\[\begin{align*}\int_{{ - 1}}^{1}{{\int_{{ - \sqrt {1 - {x^2}} }}^{0}{{\cos \left( {{x^2} + {y^2}} \right)\,dy}}\,dx}} & = \int_{\pi }^{{2\pi }}{{\left. {\frac{1}{2}\sin \left( {{r^2}} \right)} \right|_0^1\,d\theta }}\\ & = \int_{\pi }^{{2\pi }}{{\frac{1}{2}\sin \left( 1 \right)\,d\theta }}\\ & = \frac{\pi }{2}\sin \left( 1 \right)\end{align*}\]